数学分析 - Eufisky - The lost book

与$\sin n^2$类似的一些问题

1.证明: $\sum_{k=1}^n\sin k^2$无界.


参看: http://www.zhihu.com/question/29094450


2.证明\[\mathop {\lim }\limits_{n \to \infty } \frac{1}{n}\sum\limits_{k = 1}^n {\sin \sqrt k }  = 0.\]

这个极限可以从《数学分析习题课讲义》下册第 39 页的一系列习题的结论推得.
==================================================
引理1.  设级数 $\displaystyle\sum_{n=1}^\infty a_nb_n$ 收敛,如果 $b_n\searrow0,n\to\infty$, 那么有$$\lim_{n\to\infty}(a_1+a_2+\cdots+a_n)b_n=0.$$
 
证明很容易,对每一个 $n\in\mathbf N_+$, 记 $c_n=a_nb_n$, 则 $a_n=\frac{c_n}{b_n}(b_n\neq0)$, 然后对 $\displaystyle\sum_{i=1}^n\frac{c_i}{b_i}$ 作一次 Abel 变换就可以做出来了.
 
推论2. 设级数 $\displaystyle\sum_{n=1}^\infty\frac{a_n}{n}$ 收敛,那么有$$\lim_{n\to\infty}\frac{a_1+a_2+\cdots+a_n}{n}=0.$$
 
在引理1中取 $b_n=\frac{1}{n},n=1,2,\cdots$, 再利用引理1就能得到上面的推论.
 
引理3. 设函数 $f\in C^1[1,+\infty)$, 如果 $\displaystyle\int_1^\infty|f'(x)|\mathrm{d}x $收敛, 那么广义积分 $\displaystyle\int_1^\infty f(x)\mathrm{d}x $与无穷级数 $\displaystyle\sum_{n=1}^\infty f(n) $有相同的敛散性.
 
证明大概思路: 由 Newton-Leibniz 定理可知 $\lim\limits_{x\to+\infty}f(x)=A $存在且有限.如果 $A\neq0$, 则显然广义积分 $\displaystyle\int_1^\infty f(x)\mathrm{d}x$ 与无穷级数 $\displaystyle\sum_{n=1}^\infty f(n) $都发散.如果$ A=0$, 此时记$S_n=\int_1^n f(x)\mathrm{d}x,T_n=\sum_{i=1}^nf(i),n=1,2,\cdots$,则数列 $\{S_n\}$ 与广义积分 $\displaystyle\int_1^\infty f(x)\mathrm{d}x$ 敛散性相同.余下来就是证明$\lim_{m\to\infty,\atop n\to\infty}|(S_m-S_n)-(T_m-T_n)|=0$,这点很简单,注意到$ \displaystyle\int_1^\infty|f'(x)|\mathrm{d}x $收敛就行了.
 
==================================================
命题4. $\lim_{n\to\infty}\frac{1}{n}\sum_{i=1}^n\sin\sqrt i=0.$
 
由推论2,只要能够证明到级数 $\displaystyle\sum_{n=1}^\infty\frac{\sin\sqrt n}{n} $收敛就行了,再依据引理3, 只要能够证明到广义积分 $\displaystyle\int_1^\infty\frac{\sin\sqrt x}{x}\mathrm{d}x$ 收敛就可以了.而事实上又有$\int_1^\infty\frac{\sin\sqrt x}{x}\mathrm{d}x=2\int_1^\infty\frac{\sin t}{t}\mathrm{d}t$,它是收敛的,这样就得到了你的问题的证明.

多元里的两道问题

“数学是你们的选择,你们随时都可以放弃。但当数学仍是你们的选择时,就必须为此负责。”
——S.Lang对他学生上课前说的话

解题过程中碰到的几个特殊数列

1.Catalan数

设数列$\{a_n\}$满足$a_0=1$,以及

\[a_n=a_0a_{n-1}+a_1a_{n-2}+\cdots+a_{n-1}a_0,\quad n\geq1,\]

试求$a_n$的表达式.(详见周民强第二册P378)


解.令$f\left( x \right) = {a_0} + {a_1}x +  \cdots  + {a_n}{x^n} +  \cdots $,以及

\begin{align*}F\left( x \right) = &xf\left( x \right) = {a_0}x + {a_1}{x^2} +  \cdots + {a_n}{x^{n + 1}} +  \cdots \\{F^2}\left( x \right) = &a_0^2{x^2} + \left( {{a_0}{a_1} + {a_1}{a_0}} \right){x^3} +  \cdots \\&+ \left( {{a_0}{a_{n - 1}} + {a_1}{a_{n - 2}} +  \cdots  + {a_{n - 1}}{a_0}} \right){x^{n + 1}} +  \cdots \\= &{a_1}{x^2} + {a_2}{x^3} +  \cdots  + {a_n}{x^{n + 1}} +  \cdots \\= & F\left( x \right) - {a_0}x,\end{align*}
由此知$F(x)=(1-\sqrt{1-4x})/2$(注意$F(0)=0$).因为有
\[\sqrt {1 - 4x}  = 1 + C_{1/2}^1\left( { - 4x} \right) + C_{1/2}^2{\left( { - 4x} \right)^2} +  \cdots  + C_{1/2}^{n + 1}{\left( { - 4x} \right)^{n + 1}} +  \cdots ,\]
所以$F(x)$的Taylor级数展开式中的$a_n$为
\begin{align*}{a_n} &=  - \frac{1}{2}C_{1/2}^{n + 1}{\left( { - 4} \right)^{n + 1}}\\&=  - \frac{1}{2}\frac{{\frac{1}{2} \cdot \left( { - \frac{1}{2}} \right) \cdot \left( { - \frac{3}{2}} \right) \cdots \left( { - \frac{{2n - 1}}{2}} \right)}}{{n!}}{\left( { - 1} \right)^{n + 1}}{4^{n + 1}} = \frac{{C_{2n}^n}}{{n + 1}}.\end{align*}

2.Bell数

背景1.设$P(x)=a_0+a_1x+\cdots+a_mx^m$为$m$次多项式,求级数$\sum\limits_{n = 0}^\infty  {\frac{{P\left( n \right)}}{{n!}}}$的和.

事实上,

\begin{align*}{b_k} &= \sum\limits_{n = 0}^\infty  {\frac{{{n^k}}}{{n!}}}  = \sum\limits_{n = 1}^\infty  {\frac{{{n^{k - 1}}}}{{\left( {n - 1} \right)!}}}  = \sum\limits_{n = 0}^\infty  {\frac{{{{\left( {n + 1} \right)}^{k - 1}}}}{{n!}}} \\&= {b_{k - 1}} + C_{k - 1}^1{b_{k - 2}} +  \cdots  + C_{k - 1}^{k - 2}{b_1} + {b_0},\end{align*}
其中$b_0=e$.
由此得到的数叫Bell数,记为$B_n$,并且
\[B\left( x \right) = \sum\limits_{n = 0}^\infty  {\frac{{B\left( n \right)}}{{n!}}{x^n}}  = {e^{{e^x} - 1}}.\]
 
回到原题,我们有\[\sum\limits_{n = 0}^\infty  {\frac{{P\left( n \right)}}{{n!}}}  = e\sum\limits_{k = 0}^m {{a_k}{B_k}} .\]

背景2.(2005年中科院考研题)设${e^{{e^x}}} = \sum\limits_{n = 0}^\infty  {{a_n}{x^n}}$,求$a_0,a_1,a_2,a_3$,并证明$a_n\geq e{(r\ln n)}^{-n}(n>2)$,其中$r$是某个大于$e$的常数.

证.利用幂级数展开式如下:

$$\mathrm e^{\mathrm e^x} = \sum_{k=0}^\infty\frac{\mathrm e^{kx}}{k!} = \sum_{k=0}^\infty\frac 1{k!}\biggl(\sum_{n=0}^\infty\frac{(kx)^n}{n!}\biggr) = \sum_{n=0}^\infty\frac{x^n}{n!}\biggl(\sum_{k=0}^\infty\frac{k^n}{k!}\biggr),$$ 
因此有
$$a_n=\frac 1{n!}\sum_{k=0}^\infty\frac{k^n}{k!} > \frac{k^n}{n!\,k!},$$
这样对每一个 $k\geq 0 $成立.
 
以下取 $k$ 使得本题的不等式成立即可. 由于本题的 $\gamma$ 可以放大, 因此只要在等价的意义下成立即可. 这时又可以将阶乘理解为 $\Gamma$ 函数, 因此 $k$ 用非整数代入是可以的. 以下证明, 取$ \displaystyle k=\frac n{\ln n} $代入已经可以得到所要的不等式.
这时就有
$$\frac {k^n}{n!\,k!}=\frac{\displaystyle\Bigl(\frac n{\ln n}\Bigr)^n}{n!\,\displaystyle\Bigl(\frac n{\ln n}\Bigr)!} \sim\frac{\displaystyle\Bigl(\frac n{\ln n}\Bigr)^n}{\displaystyle\sqrt{2\pi n}\Bigl(\frac n{\mathrm e}\Bigr)^n\cdot \sqrt{\frac{2\pi n}{\ln n}}\Bigl(\frac n{\mathrm e\ln n}\Bigr)^{\frac n{\ln n}}} =\frac{(\mathrm e\ln n)^{\frac n{\ln n}}\sqrt{\ln n}}{2\pi n(\ln n)^n}. $$
 
由于最后一式中的分子为无穷大量, 大于$ \mathrm e $没有问题. 又由于 $a>1$ 时, $\displaystyle\frac{2\pi n}{a^n}=o(1)$, 因此任取 $\gamma>\mathrm e$, 存在 $N$, 使得当 $n>N$ 时成立 $\displaystyle a_n > \frac{\mathrm e}{(\gamma\ln n)^n}$. 最后再放大 $\gamma$ 使得不等式对一切 $n\geq 2$ 成立即可.

来源:http://www.math.org.cn/forum.php?mod=viewthread&tid=21979&extra=page%3D8

 

Euler-Maclaurin求和公式估计梯形积分公式的误差

西西在大学群里的一道题,也是2014年第六届非数竞赛预赛最后一题的推广:


设${A_n} = \frac{n}{{{n^2} + 1}} + \frac{n}{{{n^2} + {2^2}}} + \cdots + \frac{n}{{{n^2} + {n^2}}}$,求极限

\[\mathop {\lim }\limits_{n \to \infty } {n^4}\left( {\frac{1}{{24}} - n\left( {n\left( {\frac{\pi }{4} - {A_n}} \right) - \frac{1}{4}} \right)} \right).\]


这里提供个一般的方法.


Euler-Maclaurin求和公式

设函数$f\in C^{(2m+2)}[a,b],h=\frac{b-a}{n},x_i=a+ih,i=0,1,\cdots,n$,则

\begin{align*}\frac{{b - a}}{n}\sum\limits_{i = 1}^n {\frac{1}{2}\left[ {f\left( {{x_{i - 1}}} \right) + f\left( {{x_i}} \right)} \right]} - \int_a^b {f\left( x \right)dx} = &\sum\limits_{k = 1}^m {\frac{{{B_{2k}}}}{{\left( {2k} \right)!}}{h^{2k}}\left[ {{f^{\left( {2k - 1} \right)}}\left( b \right) - {f^{\left( {2k - 1} \right)}}\left( a \right)} \right]} \\&+ \frac{{{B_{2m + 2}}}}{{\left( {2m + 2} \right)!}}{h^{2m + 2}}{f^{\left( {2m + 2} \right)}}\left( \xi \right)\left( {b - a} \right),\end{align*}

其中$\xi\in [a,b]$, $B_{2k}(k=1,2,\cdots,m+1)$是Bernoulli数且$B_2=\frac16,B_4=-\frac{1}{30},B_6=\frac{1}{42}$.


enlightened解:取$a=0,b=1,f(x)=\frac{1}{1+x^2}$,则$h=\frac1n,x_i=\frac{i}{n},A_n=\frac{1}{n}\sum\limits_{i = 1}^n {f\left( {{x_i}} \right)}$,则

\begin{align*}&{A_n} + \frac{1}{{4n}} - \frac{\pi }{4} = \frac{1}{2}\left[ {\left( {{A_n} - \frac{1}{{2n}} + \frac{1}{n}} \right) + {A_n}} \right] - \frac{\pi }{4} = \frac{{{B_2}}}{{2!}} \cdot \frac{1}{{{n^2}}}\left[ {f'\left( 1 \right) - f'\left( 0 \right)} \right]\\+ &\frac{{{B_4}}}{{4!}} \cdot \frac{1}{{{n^4}}}\left[ {f'''\left( 1 \right) - f'''\left( 0 \right)} \right] + \frac{{{B_6}}}{{6!}} \cdot \frac{1}{{{n^6}}}\left[ {{f^{\left( 5 \right)}}\left( 1 \right) - {f^{\left( 5 \right)}}\left( 0 \right)} \right] + \frac{{{B_8}}}{{8!}} \cdot \frac{1}{{{n^8}}}{f^{\left( 8 \right)}}\left( \xi \right),\end{align*}

其中$\xi\in [0,1]$,也即

\[{n^4}\left( {\frac{1}{{24}} - n\left( {n\left( {\frac{\pi }{4} - {A_n}} \right) - \frac{1}{4}} \right)} \right) = \frac{1}{{2016}} + \frac{{{B_8}}}{{8!}} \cdot \frac{1}{{{n^2}}}{f^{\left( 8 \right)}}\left( \xi \right),\]

注意到${f^{\left( 8 \right)}}\left( \xi \right)$有界,因此$n\to\infty$时,所求极限为$\frac{1}{{2016}}$.

2014年浙江大学数学分析考研试题解答

6.设空间体积为$V$的任意$\Omega,X_0\in \Omega ,0<\alpha<3$.证明

\[\int_\Omega {{{\left| {X - {X_0}} \right|}^{\alpha - 3}}dX} \le C{V^{\alpha /3}}, \text{其中$C$只与$\alpha$有关}.\]


enlightened证:(Veer)由于$\alpha-3>-3$且$\displaystyle \int_\Omega {{{\left| {X - {X_0}} \right|}^{\alpha - 3}}dX}$为三重积分,故积分广义可积.

 

在$X_0$处作一以$X_0$为圆心的球$D$,使其体积为$V_D=V$.设$D$的半径为$R$.记$D_1=D\cap \Omega,D_2=D/D_1,\Omega_2=\Omega/D_1$,则易知$V_{D_2}=V_{\Omega_2}$.显然

\begin{align*}\int_D {{{\left| {X - {X_0}} \right|}^{\alpha - 3}}dX} &= \int_{{D_1}} {{{\left| {X - {X_0}} \right|}^{\alpha - 3}}dX} + \int_{{D_2}} {{{\left| {X - {X_0}} \right|}^{\alpha - 3}}dX} \\\int_\Omega {{{\left| {X - {X_0}} \right|}^{\alpha - 3}}dX} &= \int_{{D_1}} {{{\left| {X - {X_0}} \right|}^{\alpha - 3}}dX} + \int_{{\Omega _2}} {{{\left| {X - {X_0}} \right|}^{\alpha - 3}}dX} .\end{align*}

由积分中值定理有

\begin{align*}\int_{{D_2}} {{{\left| {X - {X_0}} \right|}^{\alpha - 3}}dX} &= {\left| {\xi - {X_0}} \right|^{\alpha - 3}}{V_{{D_2}}},\xi \in {D_2}\\\int_{{\Omega _2}} {{{\left| {X - {X_0}} \right|}^{\alpha - 3}}dX} &= {\left| {\eta - {X_0}} \right|^{\alpha - 3}}{V_{{\Omega_2}}},\eta \in {\Omega _2}.\end{align*}

易知${\left| {\xi - {X_0}} \right|^{\alpha - 3}} \ge {\left| {\eta - {X_0}} \right|^{\alpha - 3}}$.又因为${V_{{D_2}}} = {V_{{\Omega _2}}}$,则\[\int_\Omega {{{\left| {X - {X_0}} \right|}^{\alpha - 3}}dX} \le \int_D {{{\left| {X - {X_0}} \right|}^{\alpha - 3}}dX} .\]由球坐标变换易得\[\int_D {{{\left| {X - {X_0}} \right|}^{\alpha - 3}}dX} = \int_0^\pi {d\varphi } \int_0^{2\pi } {d\theta } \int_0^R {{r^{\alpha - 1}}\sin \varphi dr} = 4\pi \frac{{{R^\alpha }}}{\alpha }.\]又因为$\displaystyle {V_D} = V = \frac{4}{3}\pi {R^3}$,则\[\int_D {{{\left| {X - {X_0}} \right|}^{\alpha - 3}}dX} = \frac{{{3^{\alpha /3}}{{\left( {4\pi } \right)}^{1 - \alpha /3}}}}{\alpha }{V^{\alpha /3}}.\]故\[\int_\Omega {{{\left| {X - {X_0}} \right|}^{\alpha - 3}}dX} \le \frac{{{3^{\alpha /3}}{{\left( {4\pi } \right)}^{1 - \alpha /3}}}}{\alpha }{V^{\alpha /3}},\]取$\displaystyle C = \frac{{{3^{\alpha /3}}{{\left( {4\pi } \right)}^{1 - \alpha /3}}}}{\alpha }$.

 

注:从上可以看出当$\Omega=D$时不等式可取等号,故$C$是最佳的,且此题可推广到$n$维上.

 


7.$f(x)$在$[0,1]$单增,证明:

\[\mathop {\lim }\limits_{y \to + \infty } \int_0^1 {f\left( x \right)\frac{{\sin xy}}{x}dx} = \frac{\pi }{2}f\left( {{0_ + }} \right).\]


enlightened证:这是Dirichlet引理,菲赫金哥尔茨的《微积分教程》第三卷P358有详细的证明.另外,汪林的《数学分析问题研究与评注》P147上有他的推广及其证明.

 

对任意给出的$\varepsilon>0$, $\exists 0<\delta<1$,使得对于$0<t\leq \delta$,

\[0 \le g\left( t \right) - g\left( {{0_ + }} \right) < M_1\varepsilon ,\]

其中$M_1$是任意给定的常数.

 

考察积分

\begin{align*}\int_0^1 {\left[ {f\left( x \right) - f\left( {{0_ + }} \right)} \right]\frac{{\sin xy}}{x}dx} &= \left( {\int_0^\delta {} + \int_\delta ^1 {} } \right)\left[ {f\left( x \right) - f\left( {{0_ + }} \right)} \right]\frac{{\sin xy}}{x}dx\\&= {I_1} + {I_2}.\end{align*}

 

对于$I_1$,运用积分第二中值定理,我们有

\[{I_1} = \left[ {f\left( \delta \right) - f\left( {{0_ + }} \right)} \right]\int_\eta ^\delta {\frac{{\sin xy}}{x}dx} = \left[ {f\left( \delta \right) - f\left( {{0_ + }} \right)} \right]\int_{y\eta }^{y\delta } {\frac{{\sin z}}{z}dz} ,\]

 

其中第二个因子对于一切值$y$一致有界.事实上,由反常积分$\displaystyle \int_0^\infty {\frac{{\sin z}}{z}dz}$的收敛性,可见当$z\to \infty$时, $z(z\geq 0)$的连续函数$\displaystyle \int_0^z {\frac{{\sin z}}{z}dz} $有有限的极限,并且对于一切值$z$有界

\[\left| {\int_0^z {\frac{{\sin z}}{z}dz} } \right| \le L\left( L \text{为常数}\right),\]从而

\[\left| {\int_{y\eta }^{y\delta } {\frac{{\sin z}}{z}dz} } \right| = \left| {\int_0^{y\delta } {} + \int_0^{y\eta } {} } \right| \le 2L.\]

 

对于第一个因子,取$M_1=\frac{1 }{{4L}}$,则有$f\left( \delta \right) - f\left( {{0_ + }} \right) < \frac{\varepsilon }{{4L}}$.

 

 

因此\[\left| {{I_1}} \right| \le \left[ {f\left( \delta \right) - f\left( {{0_ + }} \right)} \right]\left| {\int_{y\eta }^{y\delta } {\frac{{\sin z}}{z}dz} } \right| < \frac{\varepsilon }{{4L}} \cdot 2L = \frac{\varepsilon }{2}.\]

 

至于$I_2$,由于$\displaystyle \int_\delta ^1 {\frac{{f\left( x \right) - f\left( {{0_ + }} \right)}}{x}dx} $存在,由Riemann-Lebesgue引理可知$\mathop {\lim }\limits_{y \to \infty } {I_2} = 0$,即对$\varepsilon >0,\exists M_2>0$,使得$y>M_2$时,有$\left| {{I_2}} \right| < \frac{\varepsilon }{2}$.

 

因此\[\left| {\int_0^1 {\left[ {f\left( x \right) - f\left( {{0_ + }} \right)} \right]\frac{{\sin xy}}{x}dx} } \right| \le \left| {{I_1}} \right| + \left| {{I_2}} \right| < \varepsilon .\]

即\[\mathop {\lim }\limits_{y \to + \infty } \int_0^1 {\left[ {f\left( x \right) - f\left( {{0_ + }} \right)} \right]\frac{{\sin xy}}{x}dx} = 0.\]

 

从而

\begin{align*}&\mathop {\lim }\limits_{y \to + \infty } \int_0^1 {f\left( x \right)\frac{{\sin xy}}{x}dx} = \frac{\pi }{2}f\left( {{0_ + }} \right)\\=& \mathop {\lim }\limits_{y \to + \infty } \int_0^1 {\left[ {f\left( x \right) - f\left( {{0_ + }} \right)} \right]\frac{{\sin xy}}{x}dx} + f\left( {{0_ + }} \right)\mathop {\lim }\limits_{y \to + \infty } \int_0^1 {\frac{{\sin xy}}{x}dx} \\= &0 + f\left( {{0_ + }} \right)\int_0^{ + \infty } {\frac{{\sin z}}{z}dz} = \frac{\pi }{2}f\left( {{0_ + }} \right).\end{align*}


 

幸子解答的李炯生习题

原先我们自己给李炯生习题作出的解答略显繁琐,这里贴出幸子的一些解答。


设$f(x)$是$2n+1$次多项式, $n$为正整数, $f(x)+1$被${(x-1)}^n$整除,而$f(x)-1$被${(x+1)}^n$整除,求$f(x)$.


解:由题意,存在$2u\left( x \right),2v\left( x \right) \in \mathbb{F}\left[ x \right]$,使得\[f\left( x \right) = 2u\left( x \right){\left( {x + 1} \right)^n} + 1 = 2v\left( x \right){\left( {x - 1} \right)^n} - 1.\]即得\[v\left( x \right){\left( {x - 1} \right)^n} + w\left( x \right){\left( {x + 1} \right)^n} = 1,\tag{$\ast$}\]其中$\deg v\left( x \right) = \deg w\left( x \right) = n + 1,w\left( x \right) = - u\left( x \right)$.

若$(\ast)$存在特解$v_1(x)$和$w_1(x)$,则${\left( {x + 1} \right)^n}|v\left( x \right) - {v_1}\left( x \right)$且${\left( {x - 1} \right)^n}|w\left( x \right) - {w_1}\left( x \right)$.

 

存在多项式$\alpha(x)$使得

\[\left\{ {\begin{array}{*{20}{c}}{v\left( x \right) = {v_1}\left( x \right) + {{\left( {x + 1} \right)}^n}\alpha \left( x \right)}\\{w\left( x \right) = {w_1}\left( x \right) - {{\left( {x - 1} \right)}^n}\alpha \left( x \right)}\end{array}} \right.,\]

而对于任意的$\alpha(x)\in \mathbb{F}[x]$,由

$$\left\{ {\begin{array}{*{20}{c}}{v\left( x \right) = {v_0}\left( x \right) + {{\left( {x + 1} \right)}^n}\alpha \left( x \right)}\\{w\left( x \right) = {w_0}\left( x \right) - {{\left( {x - 1} \right)}^n}\alpha \left( x \right)}\end{array}} \right.$$

定义的$v(x),w(x)$均满足$(\ast)$.由上式定义的$v(x)$和$w(x)$为$(\ast)$通解,于是$v\left( x \right) = \left[ {1 - w\left( x \right){{\left( {x + 1} \right)}^n}} \right]{\left( {x - 1} \right)^{ - n}},w\left( x \right) = \left[ {1 - v\left( x \right){{\left( {x - 1} \right)}^n}} \right]{\left( {x + 1} \right)^{ - n}},v\left( { - 1} \right) = {\left( { - 2} \right)^{ - n}},w\left( 1 \right) = {2^{ - n}}$,

 

当$1\leq i\leq n-1$时,利用Leibniz公式

\begin{align*}{v^{\left( i \right)}}\left( { - 1} \right) &= \sum\limits_{k = 0}^i {{{\left[ {1 - w\left( x \right){{\left( {x + 1} \right)}^n}} \right]}^{\left( k \right)}}{{\left[ {{{\left( {x - 1} \right)}^{ - n}}} \right]}^{\left( {i - k} \right)}}} {|_{x = - 1}} = {\left( { - 1} \right)^n}{2^{ - n - i}}n \cdots \left( {n + i - 1} \right)\\{w^{\left( i \right)}}\left( 1 \right) &= \sum\limits_{k = 0}^i {{{\left[ {1 - v\left( x \right){{\left( {x - 1} \right)}^n}} \right]}^{\left( k \right)}}{{\left[ {{{\left( {x + 1} \right)}^{ - n}}} \right]}^{\left( {i - k} \right)}}} {|_{x = 1}} = {\left( { - 1} \right)^i}{2^{ - n - i}}n \cdots \left( {n + i - 1} \right).\end{align*}

由Taylor公式,存在唯一的次数小于$n$的多项式\[{v_0}\left( x \right) = \frac{1}{{{{\left( { - 2} \right)}^n}}}\sum\limits_{i = 0}^{n - 1} {\frac{1}{{{2^i}}}C_{n + i - 1}^i{{\left( {x + 1} \right)}^i}} ,{w_0}\left( x \right) = \frac{1}{{{2^n}}}\sum\limits_{i = 0}^{n - 1} {\frac{1}{{{{\left( { - 2} \right)}^i}}}C_{n + i - 1}^i{{\left( {x - 1} \right)}^i}} \]使得$(\ast)$成立.

 

因此对任意的一次多项式$\alpha \left( x \right)$,

 

$f\left( x \right) = 2\left[ {{{\left( {x - 1} \right)}^n}\alpha \left( x \right) - {w_0}\left( x \right)} \right]{\left( {x + 1} \right)^n} + 1= 2\left[ {{v_0}\left( x \right) + {{\left( {x + 1} \right)}^n}\alpha \left( x \right)} \right]{\left( {x - 1} \right)^n} - 1$均满足题设.


设$A\in \mathbb{R}^{2n\times 2n}$,且$A\left( {\begin{array}{*{20}{c}}0&{{I_{\left( n \right)}}}\\{ - {I_{\left( n \right)}}}&0\end{array}} \right){A^T} = \left( {\begin{array}{*{20}{c}}0&{{I_{\left( n \right)}}}\\{ - {I_{\left( n \right)}}}&0\end{array}} \right)$.证明: $\det A=1$.


证:令$U = \frac{1}{{\sqrt 2 }}\left( {\begin{array}{*{20}{c}}{{I_n}}&{ - i{I_n}}\\{{I_n}}&{i{I_n}}\end{array}} \right),A = \left( {\begin{array}{*{20}{c}}B&C\\D&G\end{array}} \right)$,

\[T = UA{U^H} = \frac{1}{2}\left( {\begin{array}{*{20}{c}}{B + G + \left( {C - D} \right)i}&{B - G - \left( {C + D} \right)i}\\{B - G + \left( {C + D} \right)i}&{B + G - \left( {C - D} \right)i}\end{array}} \right) = \left( {\begin{array}{*{20}{c}}P&Q\\{\bar Q}&{\bar P}\end{array}} \right).\]

由$A\left( {\begin{array}{*{20}{c}}O&{{I_n}}\\{ - {I_n}}&O\end{array}} \right){A^T} = \left( {\begin{array}{*{20}{c}}O&{{I_n}}\\{{I_n}}&O\end{array}} \right)$知

\[\left( {\begin{array}{*{20}{c}}P&Q\\{\bar Q}&{\bar P}\end{array}} \right)\left( {\begin{array}{*{20}{c}}{{I_n}}&O\\O&{ - {I_n}}\end{array}} \right)\left( {\begin{array}{*{20}{c}}{{P^H}}&{{Q^T}}\\{{Q^H}}&{{P^T}}\end{array}} \right) = \left( {\begin{array}{*{20}{c}}{{I_n}}&O\\O&{ - {I_n}}\end{array}} \right).\]

即$P{P^H} - Q{Q^H} = {I_n}$及$P{Q^T} = Q{P^T}$,注意到$P{P^H} = {I_n} + Q{Q^H} > 0$,即$\det P\neq0$,且$\det A=\det T$.而$\det \left( {\begin{array}{*{20}{c}}P&Q\\{\bar Q}&{\bar P}\end{array}} \right)\left( {\begin{array}{*{20}{c}}{{P^H}}&O\\{ - {Q^H}}&{{I_n}}\end{array}} \right) = \det \left( {\begin{array}{*{20}{c}}{{I_n}}&Q\\O&{\bar P}\end{array}} \right)$.因此$\det A=1$.


 

曲线积分的计算

张元博问了这么一道题:计算曲线积分\[\oint_C {\left( {{{\left( {x + 1} \right)}^2} + {{\left( {y - 2} \right)}^2}} \right)ds} ,\]其中$C$表示曲面$x^2+y^2+z^2=1$与$x+y+z=1$的交线.


解:我是利用常规的三角换元解决的.联立方程有

\[\left\{ \begin{array}{l}{x^2} + {y^2} + {z^2} = 1\\x + y + z = 1\end{array} \right. \Rightarrow {\left( {x - \frac{1}{2} + \frac{y}{2}} \right)^2} + \frac{3}{4}{\left( {y - \frac{1}{3}} \right)^2} = \frac{1}{3}.\]
\[\left\{ \begin{array}{l}x - \frac{1}{2} + \frac{y}{2} = \frac{1}{{\sqrt 3 }}\cos \theta \\\frac{{\sqrt 3 }}{2}\left( {y - \frac{1}{3}} \right) = \frac{1}{{\sqrt 3 }}\sin \theta \end{array} \right. \Rightarrow \left\{ \begin{array}{l}x = \frac{1}{3} - \frac{1}{3}\sin \theta  + \frac{1}{{\sqrt 3 }}\cos \theta \\y = \frac{1}{3} + \frac{2}{3}\sin \theta \\z = 1 - x - y = \frac{1}{3} - \frac{1}{3}\sin \theta  - \frac{1}{{\sqrt 3 }}\cos \theta \end{array} \right.\]
则\[ds = \sqrt {{{\left( {x'\left( \theta  \right)} \right)}^2} + {{\left( {y'\left( \theta  \right)} \right)}^2} + {{\left( {z'\left( \theta  \right)} \right)}^2}}  = \frac{{\sqrt 6 }}{3}.\]
因此
\begin{align*}&\oint_C {\left( {{{\left( {x + 1} \right)}^2} + {{\left( {y - 2} \right)}^2}} \right)ds} \\= &\frac{{\sqrt 6 }}{3}\int_0^{2\pi } {\left( {\frac{5}{9}{{\sin }^2}\theta  + \frac{1}{3}{{\cos }^2}\theta  - \frac{{28}}{9}\sin \theta  + \frac{8}{{3\sqrt 3 }}\cos \theta  - \frac{2}{{3\sqrt 3 }}\sin \theta \cos \theta  + \frac{{41}}{9}} \right)d\theta } \\= &\frac{{\sqrt 6 }}{3} \times 5 = \frac{{5\sqrt 6 }}{3}.\end{align*}

 

余神题解

二重积分难题荟萃

第一个是09年西北大学的考研题,也在史济怀老师的数分书上找得到.这里的解答就是史老爷子自己的.


计算积分\[\iint_D {\frac{1}{{xy\left( {\ln^2 x + {{\ln }^2}y} \right)}}} dxdy,D = \left\{ {\left( {x,y} \right)\left| {x + y \ge 1,{x^2} + {y^2} \le 1} \right.} \right\}.\]


enlightened解.作变换$x=e^{r\cos\theta},y=e^{r\sin\theta}$,则

\[\frac{{\partial \left( {x,y} \right)}}{{\partial \left( {r,\theta } \right)}} = \left| {\begin{array}{*{20}{c}}{\cos \theta {e^{r\cos \theta }}}&{ - r\sin \theta {e^{r\cos \theta }}}\\{\sin \theta {e^{r\sin \theta }}}&{r\cos \theta {e^{r\sin \theta }}}\end{array}} \right| = r{e^{r\sin \theta }}{e^{r\cos \theta }}.\]

原积分变为\[I = \iint_\Delta {\frac{{drd\theta }}{r}} .\]

这里的$\Delta$是变换以后的积分区域.注意$x+y=1$和$x^2+y^2=1$分别被变为

\[\left\{ \begin{array}{l}{e^{r\cos \theta }} + {e^{r\sin \theta }} = 1,\\{e^{2r\cos \theta }} + {e^{2r\sin \theta }} = 1.\end{array} \right.\]

 

现在来分析由上述两条曲线所围成的$(r,\theta)$平面上的区域是什么形状.从第一个式子可以看出$\theta$的变化范围必须使$\cos \theta$和$\sin \theta$都取负值,故$\theta$只能在$\left[\pi,\frac32\pi \right]$中取值.

 

假设由第一个式子确定的函数为$r=r(\theta)$,则由第二个式子确定的函数便为$r=\frac12 r(\theta)$.因此

 

\[I = \iint_\Delta {\frac{{drd\theta }}{r}} = \int_\pi ^{\frac{3}{2}\pi } {d\theta } \int_{\frac{1}{2}r\left( \theta \right)}^{r\left( \theta \right)} {\frac{{dr}}{r}} = \frac{\pi }{2}\ln 2.\]

 

苹果公司总部


 

2015年浙大数分题

1. 求 $\lim\limits_{n\rightarrow+\infty} \dfrac{(n^2+1)(n^2+2)..(n^2+n)}{(n^2-1)(n^2-2)..(n^2-n)}$.
 
2. 求$\displaystyle\lim\limits_{x \rightarrow 0+} \dfrac{1}{x^5}\int_0^x e^{-t^2}dt+\dfrac{1}{3}\dfrac{1}{x^2}-\dfrac{1}{x^4}$.
 
3. $\displaystyle I(r)=\oint \dfrac{y}{x^2+y^2}\mathrm{d}x-\frac{x}{x^2+y^2}\mathrm{d}y$, 其中曲线方程为$x^2+y^2+xy=r^2$, 取正方向, 求$\lim\limits_{r\rightarrow\infty}I(r)$.
 
4. 求$\displaystyle\int_{e^{-2n\pi}}^0 \sin\ln\dfrac{1}{x}\mathrm{d}x$.
 
5. 考察黎曼函数的连续性, 可微性, 黎曼可积性.
 
6. 在$\mathbb{R}^n$中, $f$为定义在某个区域上的一个函数, 有一阶连续偏导, 且偏导数有界. 
证明:
(1) 若$D$为凸区域证明$f$一致连续. 
(2)考察$D$不是凸区域的情况.
 
7. $\{f_n\}$为一个连续函数列, 且对于任意给定的$x$, $\{f_n(x)\}$有界, 证明存在一个小区间在此小区间内$f_n$一致有界.
 
8. (1) 证明$\Gamma(s)$在 $(0,\infty$内无穷次可微.
(2) 证明$\Gamma(s)$ , $\ln(\Gamma)$都是严格凸函数.
 
9. $f$ 二阶可微, 且$f$, $f'$, $f''$ 都大于等于$0$, 且存在一个正数$c$, $f''(x)\leq cf(x)$. 证明:
(1) $\displaystyle\lim\limits_{x\rightarrow-\infty}f(x)=0$;
(2) 证明存在正数$a$, 有$f'\leq af$,并求出$a$.
 
10. 证明 Fejer定理.
 
11. 设$f$在$[A, B]$上黎曼可积, $0<f<1$, 对于任意的$\varepsilon$, 构造一个函数$g$, 满足
(1) $g$是一个阶梯函数, 且取值只能为$0$或$1$.
(2) $\displaystyle\left| \int_a^b f-g \mathrm{d}x\right|<\varepsilon$, $a$, $b$ 属于$[A,B]$不等号关于$a$, $b$是一致的.